15
$\begingroup$

This question was asked earlier on math.stackexchange: click here. See the comments and the answer by Jack Schmidt there.

Let $M$ be a module over a commutative ring $R$.

It is possible that $M \otimes M = 0$ if $M$ is nonzero, for example when $R = \mathbb{Z}$ and $M = \mathbb{Q}/ \mathbb{Z}$.

What about when higher tensor powers of $M$ are zero? If $M \otimes M \otimes M = 0$, is it possible that $M \otimes M$ is nonzero? More generally if $M^{\otimes n} = 0$ for $n \geq 3$, is it possible that $M^{\otimes n-1}$ is nonzero?

The answer by Jack Schmidt in the math.stackexchange question shows that the answer to these questions is no, in the case of $R = \mathbb{Z}$, ie. in the case of abelian groups. Quote from his answer:

Theorem: If $M$ is an abelian group with $M \otimes M \otimes M = 0$, then $M$ is a divisible torsion abelian group and $M \otimes M = 0$.

$\endgroup$

1 Answer 1

10
$\begingroup$

My previous attempt was completely wrong, as Jason Starr politely pointed out.

But I think the idea I was grasping for does work, in this example:

Let $R=k[x,y]$ for a field $k$, and let $$M=\frac{k[x,y,y^{-1}]}{k[x,y]}\oplus\frac{k[x,x^{-1},y]}{k[x,y]}.$$

[So that Jason's comment still makes sense, I'll leave my previous stupid wrong answer:

How about $R=\mathbb{Z}\times\mathbb{Z}$ and $M=(\mathbb{Q}/\mathbb{Z}\times\mathbb{Z})\oplus(\mathbb{Z}\times\mathbb{Q}/\mathbb{Z})$?]

$\endgroup$
3
  • 3
    $\begingroup$ That doesn't work. There is a surjective homomorphism to $R=\mathbb{Z}\times \mathbb{Z}$, namely multiplication by $(0,1)$ on the first summand plus multiplication by $(1,0)$ on the second summand. Since tensor product over $R$ is right exact, for every integer $n>0$, there is a surjectiction $M^{\otimes n} \to R^{\otimes n} = R$. $\endgroup$ May 9, 2014 at 12:52
  • $\begingroup$ @JasonStarr: You're quite right, of course. I was being stupid. But I think the idea I was trying to make into an example does work, and I've edited my answer. $\endgroup$ May 9, 2014 at 13:43
  • 1
    $\begingroup$ I believe the new example works. By using $n$ variables instead of $2$ variables, you should also get a module $M$ such that $M^{\otimes n}$ is nonzero, yet $M^{\otimes(n+1)}$ is zero. $\endgroup$ May 9, 2014 at 18:14

Your Answer

By clicking “Post Your Answer”, you agree to our terms of service and acknowledge you have read our privacy policy.

Not the answer you're looking for? Browse other questions tagged or ask your own question.